Statistics (12th Edition)

Published by Pearson
ISBN 10: 0321755936
ISBN 13: 978-0-32175-593-3

Chapter 9 - Inferences Based on Two Samples - Exercises 9.30 - 9.50 - Learning the Mechanics - Page 436: 9.36c

Answer

Confidence interval: -4.9831 to -2.4169

Work Step by Step

df = n - 1 = 10 - 1 = 9 $t_{α/2} = 0.05$ From student's t distribution table, we have: $t_{0.05} = 1.833$ $E = t. \frac{s_{d}}{n} = 1.833 . \frac{2.2136}{\sqrt 10}$ $ = 1.833 . 0.7 = 1.283$1$ Confidence interval will be: $\bar d - E = -3.7 - 1.2831 = -4.9831$ $\bar d - E = -3.7 + 1.2831 = -2.4169$
Update this answer!

You can help us out by revising, improving and updating this answer.

Update this answer

After you claim an answer you’ll have 24 hours to send in a draft. An editor will review the submission and either publish your submission or provide feedback.